“Quick” proof of the fundamental lemma of calculus of variationsWhy can't we construct a counter-example to the Fundamental Lemma of the Calculus of Variations?Fundamental lemma of calculus of variation .A variation of fundamental lemma of variation of calculus .Variation of the fundamental lemma of calculus of variationProof of fundamental lemma of calculus of variation.Proof of Fundamental Lemma of Calculus of VariationsFundamental lemma of calculus of variation, about hypothesisWeakening the Fundamental Lemma of Calculus of VariationsFundamental Lemma for variational calculus for measurable functionCompact support in calculus of variations

How to preserve electronics (computers, ipads, phones) for hundreds of years?

Should a narrator ever describe things based on a character's view instead of facts?

C++ lambda syntax

Mortal danger in mid-grade literature

Why is indicated airspeed rather than ground speed used during the takeoff roll?

Why is implicit conversion not ambiguous for non-primitive types?

How do I lift the insulation blower into the attic?

Why is "la Gestapo" feminine?

categorizing a variable turns it from insignificant to significant

Taking the numerator and the denominator

Pre-Employment Background Check With Consent For Future Checks

Unfrosted light bulb

Why do Radio Buttons not fill the entire outer circle?

Highest stage count that are used one right after the other?

Rendered textures different to 3D View

What is the meaning of "You've never met a graph you didn't like?"

Can you take a "free object interaction" while incapacitated?

Reason why a kingside attack is not justified

Can a Knock spell open the door to Mordenkainen's Magnificent Mansion?

Air travel with refrigerated insulin

Friend wants my recommendation but I don't want to give it to him

Extract substring according to regexp with sed or grep

Reasons for having MCU pin-states default to pull-up/down out of reset

How do I prevent inappropriate ads from appearing in my game?



“Quick” proof of the fundamental lemma of calculus of variations


Why can't we construct a counter-example to the Fundamental Lemma of the Calculus of Variations?Fundamental lemma of calculus of variation .A variation of fundamental lemma of variation of calculus .Variation of the fundamental lemma of calculus of variationProof of fundamental lemma of calculus of variation.Proof of Fundamental Lemma of Calculus of VariationsFundamental lemma of calculus of variation, about hypothesisWeakening the Fundamental Lemma of Calculus of VariationsFundamental Lemma for variational calculus for measurable functionCompact support in calculus of variations













1












$begingroup$


Here's the statement:




Let $f in C([a,b])$ and $H$ be the set $hin C([a,b]):h(a)=h(b)=0$. If $int_a^bf(x)h(x),textdx=0$ for all $hin H$, then $f(x)=0$ for all $xin [a,b]$.




I saw a lot of long proofs for this but I thought I could do better, but I think there could be a subtle error. Here's my attempt:



Consider the constant $c$ and the function $phi$ defined as



$$phi(x)=int_a^xh(x)-c,textdx, c=frac1b-aint_a^bh(x),textdx$$



obviously $phi(a)=phi(b)=0$, hence $phi in H$. Also, $h$ is integrable because it is continuous in $[a,b]$.



By hypothesis, we have



$$int_a^bf(x)h(x),textdx=0Leftrightarrowint_a^bf(x)phi'(x),textdx=0$$



Then we can use Du Bois-Reymond's lemma, which states




Let $H$ be the set $hin C^1([a,b]):h(a)=h(b)=0$. If $fin C([a,b])$ and $int_a^b f(x)h'(x),textdx=0$ for all $hin H$, then $f(x)$ is constant for all $xin[a,b]$.




The lemma can be used directly to get that $f(x)=k$, where $kinmathbbR$.



Then our hypothesis is simply



$$int_a^bkh(x),textdx=0$$



for this to be true for all $h in H$, we must have $k=0$, because if $h(x)$ were, for example, a positive function in $(a,b)$ with $h(a)=h(b)=0$, it would not be true... thus the lemma is proved.



Now, where does it all fall apart?










share|cite|improve this question









$endgroup$











  • $begingroup$
    Don't you need the first statement to prove the lemma? It looks like the lemma follows from the first using integration by parts (do you know any other way to deduce it? because otherwise it is a circular proof).
    $endgroup$
    – Yanko
    Mar 13 at 18:31











  • $begingroup$
    The proof is very similar. You cannot use integration by parts if the function $f$ in Du Bois-Raymond's lemma is not in $C^1([a,b])$, but it is provable without it. Essentially, I'm construction the function $phi$ because it is in $C^1([a,b])$, where I can apply Du-Bois Raymond ($h$ is not $C^1([a,b])$ in the fundamental lemma).
    $endgroup$
    – AstlyDichrar
    Mar 13 at 18:38











  • $begingroup$
    To add on to this, there is a proof for Du Bois-Reymond that does not depend on any of what I did.
    $endgroup$
    – AstlyDichrar
    Mar 13 at 18:45











  • $begingroup$
    I see, well in that case I don't see any flaws with your proof.
    $endgroup$
    – Yanko
    Mar 13 at 18:46















1












$begingroup$


Here's the statement:




Let $f in C([a,b])$ and $H$ be the set $hin C([a,b]):h(a)=h(b)=0$. If $int_a^bf(x)h(x),textdx=0$ for all $hin H$, then $f(x)=0$ for all $xin [a,b]$.




I saw a lot of long proofs for this but I thought I could do better, but I think there could be a subtle error. Here's my attempt:



Consider the constant $c$ and the function $phi$ defined as



$$phi(x)=int_a^xh(x)-c,textdx, c=frac1b-aint_a^bh(x),textdx$$



obviously $phi(a)=phi(b)=0$, hence $phi in H$. Also, $h$ is integrable because it is continuous in $[a,b]$.



By hypothesis, we have



$$int_a^bf(x)h(x),textdx=0Leftrightarrowint_a^bf(x)phi'(x),textdx=0$$



Then we can use Du Bois-Reymond's lemma, which states




Let $H$ be the set $hin C^1([a,b]):h(a)=h(b)=0$. If $fin C([a,b])$ and $int_a^b f(x)h'(x),textdx=0$ for all $hin H$, then $f(x)$ is constant for all $xin[a,b]$.




The lemma can be used directly to get that $f(x)=k$, where $kinmathbbR$.



Then our hypothesis is simply



$$int_a^bkh(x),textdx=0$$



for this to be true for all $h in H$, we must have $k=0$, because if $h(x)$ were, for example, a positive function in $(a,b)$ with $h(a)=h(b)=0$, it would not be true... thus the lemma is proved.



Now, where does it all fall apart?










share|cite|improve this question









$endgroup$











  • $begingroup$
    Don't you need the first statement to prove the lemma? It looks like the lemma follows from the first using integration by parts (do you know any other way to deduce it? because otherwise it is a circular proof).
    $endgroup$
    – Yanko
    Mar 13 at 18:31











  • $begingroup$
    The proof is very similar. You cannot use integration by parts if the function $f$ in Du Bois-Raymond's lemma is not in $C^1([a,b])$, but it is provable without it. Essentially, I'm construction the function $phi$ because it is in $C^1([a,b])$, where I can apply Du-Bois Raymond ($h$ is not $C^1([a,b])$ in the fundamental lemma).
    $endgroup$
    – AstlyDichrar
    Mar 13 at 18:38











  • $begingroup$
    To add on to this, there is a proof for Du Bois-Reymond that does not depend on any of what I did.
    $endgroup$
    – AstlyDichrar
    Mar 13 at 18:45











  • $begingroup$
    I see, well in that case I don't see any flaws with your proof.
    $endgroup$
    – Yanko
    Mar 13 at 18:46













1












1








1





$begingroup$


Here's the statement:




Let $f in C([a,b])$ and $H$ be the set $hin C([a,b]):h(a)=h(b)=0$. If $int_a^bf(x)h(x),textdx=0$ for all $hin H$, then $f(x)=0$ for all $xin [a,b]$.




I saw a lot of long proofs for this but I thought I could do better, but I think there could be a subtle error. Here's my attempt:



Consider the constant $c$ and the function $phi$ defined as



$$phi(x)=int_a^xh(x)-c,textdx, c=frac1b-aint_a^bh(x),textdx$$



obviously $phi(a)=phi(b)=0$, hence $phi in H$. Also, $h$ is integrable because it is continuous in $[a,b]$.



By hypothesis, we have



$$int_a^bf(x)h(x),textdx=0Leftrightarrowint_a^bf(x)phi'(x),textdx=0$$



Then we can use Du Bois-Reymond's lemma, which states




Let $H$ be the set $hin C^1([a,b]):h(a)=h(b)=0$. If $fin C([a,b])$ and $int_a^b f(x)h'(x),textdx=0$ for all $hin H$, then $f(x)$ is constant for all $xin[a,b]$.




The lemma can be used directly to get that $f(x)=k$, where $kinmathbbR$.



Then our hypothesis is simply



$$int_a^bkh(x),textdx=0$$



for this to be true for all $h in H$, we must have $k=0$, because if $h(x)$ were, for example, a positive function in $(a,b)$ with $h(a)=h(b)=0$, it would not be true... thus the lemma is proved.



Now, where does it all fall apart?










share|cite|improve this question









$endgroup$




Here's the statement:




Let $f in C([a,b])$ and $H$ be the set $hin C([a,b]):h(a)=h(b)=0$. If $int_a^bf(x)h(x),textdx=0$ for all $hin H$, then $f(x)=0$ for all $xin [a,b]$.




I saw a lot of long proofs for this but I thought I could do better, but I think there could be a subtle error. Here's my attempt:



Consider the constant $c$ and the function $phi$ defined as



$$phi(x)=int_a^xh(x)-c,textdx, c=frac1b-aint_a^bh(x),textdx$$



obviously $phi(a)=phi(b)=0$, hence $phi in H$. Also, $h$ is integrable because it is continuous in $[a,b]$.



By hypothesis, we have



$$int_a^bf(x)h(x),textdx=0Leftrightarrowint_a^bf(x)phi'(x),textdx=0$$



Then we can use Du Bois-Reymond's lemma, which states




Let $H$ be the set $hin C^1([a,b]):h(a)=h(b)=0$. If $fin C([a,b])$ and $int_a^b f(x)h'(x),textdx=0$ for all $hin H$, then $f(x)$ is constant for all $xin[a,b]$.




The lemma can be used directly to get that $f(x)=k$, where $kinmathbbR$.



Then our hypothesis is simply



$$int_a^bkh(x),textdx=0$$



for this to be true for all $h in H$, we must have $k=0$, because if $h(x)$ were, for example, a positive function in $(a,b)$ with $h(a)=h(b)=0$, it would not be true... thus the lemma is proved.



Now, where does it all fall apart?







proof-verification calculus-of-variations






share|cite|improve this question













share|cite|improve this question











share|cite|improve this question




share|cite|improve this question










asked Mar 13 at 18:26









AstlyDichrarAstlyDichrar

42248




42248











  • $begingroup$
    Don't you need the first statement to prove the lemma? It looks like the lemma follows from the first using integration by parts (do you know any other way to deduce it? because otherwise it is a circular proof).
    $endgroup$
    – Yanko
    Mar 13 at 18:31











  • $begingroup$
    The proof is very similar. You cannot use integration by parts if the function $f$ in Du Bois-Raymond's lemma is not in $C^1([a,b])$, but it is provable without it. Essentially, I'm construction the function $phi$ because it is in $C^1([a,b])$, where I can apply Du-Bois Raymond ($h$ is not $C^1([a,b])$ in the fundamental lemma).
    $endgroup$
    – AstlyDichrar
    Mar 13 at 18:38











  • $begingroup$
    To add on to this, there is a proof for Du Bois-Reymond that does not depend on any of what I did.
    $endgroup$
    – AstlyDichrar
    Mar 13 at 18:45











  • $begingroup$
    I see, well in that case I don't see any flaws with your proof.
    $endgroup$
    – Yanko
    Mar 13 at 18:46
















  • $begingroup$
    Don't you need the first statement to prove the lemma? It looks like the lemma follows from the first using integration by parts (do you know any other way to deduce it? because otherwise it is a circular proof).
    $endgroup$
    – Yanko
    Mar 13 at 18:31











  • $begingroup$
    The proof is very similar. You cannot use integration by parts if the function $f$ in Du Bois-Raymond's lemma is not in $C^1([a,b])$, but it is provable without it. Essentially, I'm construction the function $phi$ because it is in $C^1([a,b])$, where I can apply Du-Bois Raymond ($h$ is not $C^1([a,b])$ in the fundamental lemma).
    $endgroup$
    – AstlyDichrar
    Mar 13 at 18:38











  • $begingroup$
    To add on to this, there is a proof for Du Bois-Reymond that does not depend on any of what I did.
    $endgroup$
    – AstlyDichrar
    Mar 13 at 18:45











  • $begingroup$
    I see, well in that case I don't see any flaws with your proof.
    $endgroup$
    – Yanko
    Mar 13 at 18:46















$begingroup$
Don't you need the first statement to prove the lemma? It looks like the lemma follows from the first using integration by parts (do you know any other way to deduce it? because otherwise it is a circular proof).
$endgroup$
– Yanko
Mar 13 at 18:31





$begingroup$
Don't you need the first statement to prove the lemma? It looks like the lemma follows from the first using integration by parts (do you know any other way to deduce it? because otherwise it is a circular proof).
$endgroup$
– Yanko
Mar 13 at 18:31













$begingroup$
The proof is very similar. You cannot use integration by parts if the function $f$ in Du Bois-Raymond's lemma is not in $C^1([a,b])$, but it is provable without it. Essentially, I'm construction the function $phi$ because it is in $C^1([a,b])$, where I can apply Du-Bois Raymond ($h$ is not $C^1([a,b])$ in the fundamental lemma).
$endgroup$
– AstlyDichrar
Mar 13 at 18:38





$begingroup$
The proof is very similar. You cannot use integration by parts if the function $f$ in Du Bois-Raymond's lemma is not in $C^1([a,b])$, but it is provable without it. Essentially, I'm construction the function $phi$ because it is in $C^1([a,b])$, where I can apply Du-Bois Raymond ($h$ is not $C^1([a,b])$ in the fundamental lemma).
$endgroup$
– AstlyDichrar
Mar 13 at 18:38













$begingroup$
To add on to this, there is a proof for Du Bois-Reymond that does not depend on any of what I did.
$endgroup$
– AstlyDichrar
Mar 13 at 18:45





$begingroup$
To add on to this, there is a proof for Du Bois-Reymond that does not depend on any of what I did.
$endgroup$
– AstlyDichrar
Mar 13 at 18:45













$begingroup$
I see, well in that case I don't see any flaws with your proof.
$endgroup$
– Yanko
Mar 13 at 18:46




$begingroup$
I see, well in that case I don't see any flaws with your proof.
$endgroup$
– Yanko
Mar 13 at 18:46










1 Answer
1






active

oldest

votes


















0












$begingroup$

Alright, my professor pointed out a possible flaw in the proof: is it really possible to represent every function in $H = h in C^1([a,b]):h(a)=h(b)=0$ as $phi(x)$, given its definition in the original post?



I'd guess that means we need to verify whether or not for every function $hin H$ ($H$ in the sense of the original definition, where we consider functions in $C([a,b])$, there is a correspondent function $fin H$ (in the $C^1([a,b])$ sense) such that $f(x)=int_a^b h(x)-c,textdx$. Somewhat informally, we need to verify if this integral is a "surjection", I think. This is probably not true, but I don't think I know enough to prove it.






share|cite|improve this answer









$endgroup$












    Your Answer





    StackExchange.ifUsing("editor", function ()
    return StackExchange.using("mathjaxEditing", function ()
    StackExchange.MarkdownEditor.creationCallbacks.add(function (editor, postfix)
    StackExchange.mathjaxEditing.prepareWmdForMathJax(editor, postfix, [["$", "$"], ["\\(","\\)"]]);
    );
    );
    , "mathjax-editing");

    StackExchange.ready(function()
    var channelOptions =
    tags: "".split(" "),
    id: "69"
    ;
    initTagRenderer("".split(" "), "".split(" "), channelOptions);

    StackExchange.using("externalEditor", function()
    // Have to fire editor after snippets, if snippets enabled
    if (StackExchange.settings.snippets.snippetsEnabled)
    StackExchange.using("snippets", function()
    createEditor();
    );

    else
    createEditor();

    );

    function createEditor()
    StackExchange.prepareEditor(
    heartbeatType: 'answer',
    autoActivateHeartbeat: false,
    convertImagesToLinks: true,
    noModals: true,
    showLowRepImageUploadWarning: true,
    reputationToPostImages: 10,
    bindNavPrevention: true,
    postfix: "",
    imageUploader:
    brandingHtml: "Powered by u003ca class="icon-imgur-white" href="https://imgur.com/"u003eu003c/au003e",
    contentPolicyHtml: "User contributions licensed under u003ca href="https://creativecommons.org/licenses/by-sa/3.0/"u003ecc by-sa 3.0 with attribution requiredu003c/au003e u003ca href="https://stackoverflow.com/legal/content-policy"u003e(content policy)u003c/au003e",
    allowUrls: true
    ,
    noCode: true, onDemand: true,
    discardSelector: ".discard-answer"
    ,immediatelyShowMarkdownHelp:true
    );



    );













    draft saved

    draft discarded


















    StackExchange.ready(
    function ()
    StackExchange.openid.initPostLogin('.new-post-login', 'https%3a%2f%2fmath.stackexchange.com%2fquestions%2f3146979%2fquick-proof-of-the-fundamental-lemma-of-calculus-of-variations%23new-answer', 'question_page');

    );

    Post as a guest















    Required, but never shown

























    1 Answer
    1






    active

    oldest

    votes








    1 Answer
    1






    active

    oldest

    votes









    active

    oldest

    votes






    active

    oldest

    votes









    0












    $begingroup$

    Alright, my professor pointed out a possible flaw in the proof: is it really possible to represent every function in $H = h in C^1([a,b]):h(a)=h(b)=0$ as $phi(x)$, given its definition in the original post?



    I'd guess that means we need to verify whether or not for every function $hin H$ ($H$ in the sense of the original definition, where we consider functions in $C([a,b])$, there is a correspondent function $fin H$ (in the $C^1([a,b])$ sense) such that $f(x)=int_a^b h(x)-c,textdx$. Somewhat informally, we need to verify if this integral is a "surjection", I think. This is probably not true, but I don't think I know enough to prove it.






    share|cite|improve this answer









    $endgroup$

















      0












      $begingroup$

      Alright, my professor pointed out a possible flaw in the proof: is it really possible to represent every function in $H = h in C^1([a,b]):h(a)=h(b)=0$ as $phi(x)$, given its definition in the original post?



      I'd guess that means we need to verify whether or not for every function $hin H$ ($H$ in the sense of the original definition, where we consider functions in $C([a,b])$, there is a correspondent function $fin H$ (in the $C^1([a,b])$ sense) such that $f(x)=int_a^b h(x)-c,textdx$. Somewhat informally, we need to verify if this integral is a "surjection", I think. This is probably not true, but I don't think I know enough to prove it.






      share|cite|improve this answer









      $endgroup$















        0












        0








        0





        $begingroup$

        Alright, my professor pointed out a possible flaw in the proof: is it really possible to represent every function in $H = h in C^1([a,b]):h(a)=h(b)=0$ as $phi(x)$, given its definition in the original post?



        I'd guess that means we need to verify whether or not for every function $hin H$ ($H$ in the sense of the original definition, where we consider functions in $C([a,b])$, there is a correspondent function $fin H$ (in the $C^1([a,b])$ sense) such that $f(x)=int_a^b h(x)-c,textdx$. Somewhat informally, we need to verify if this integral is a "surjection", I think. This is probably not true, but I don't think I know enough to prove it.






        share|cite|improve this answer









        $endgroup$



        Alright, my professor pointed out a possible flaw in the proof: is it really possible to represent every function in $H = h in C^1([a,b]):h(a)=h(b)=0$ as $phi(x)$, given its definition in the original post?



        I'd guess that means we need to verify whether or not for every function $hin H$ ($H$ in the sense of the original definition, where we consider functions in $C([a,b])$, there is a correspondent function $fin H$ (in the $C^1([a,b])$ sense) such that $f(x)=int_a^b h(x)-c,textdx$. Somewhat informally, we need to verify if this integral is a "surjection", I think. This is probably not true, but I don't think I know enough to prove it.







        share|cite|improve this answer












        share|cite|improve this answer



        share|cite|improve this answer










        answered 7 hours ago









        AstlyDichrarAstlyDichrar

        42248




        42248



























            draft saved

            draft discarded
















































            Thanks for contributing an answer to Mathematics Stack Exchange!


            • Please be sure to answer the question. Provide details and share your research!

            But avoid


            • Asking for help, clarification, or responding to other answers.

            • Making statements based on opinion; back them up with references or personal experience.

            Use MathJax to format equations. MathJax reference.


            To learn more, see our tips on writing great answers.




            draft saved


            draft discarded














            StackExchange.ready(
            function ()
            StackExchange.openid.initPostLogin('.new-post-login', 'https%3a%2f%2fmath.stackexchange.com%2fquestions%2f3146979%2fquick-proof-of-the-fundamental-lemma-of-calculus-of-variations%23new-answer', 'question_page');

            );

            Post as a guest















            Required, but never shown





















































            Required, but never shown














            Required, but never shown












            Required, but never shown







            Required, but never shown

































            Required, but never shown














            Required, but never shown












            Required, but never shown







            Required, but never shown







            Popular posts from this blog

            Lowndes Grove History Architecture References Navigation menu32°48′6″N 79°57′58″W / 32.80167°N 79.96611°W / 32.80167; -79.9661132°48′6″N 79°57′58″W / 32.80167°N 79.96611°W / 32.80167; -79.9661178002500"National Register Information System"Historic houses of South Carolina"Lowndes Grove""+32° 48' 6.00", −79° 57' 58.00""Lowndes Grove, Charleston County (260 St. Margaret St., Charleston)""Lowndes Grove"The Charleston ExpositionIt Happened in South Carolina"Lowndes Grove (House), Saint Margaret Street & Sixth Avenue, Charleston, Charleston County, SC(Photographs)"Plantations of the Carolina Low Countrye

            random experiment with two different functions on unit interval Announcing the arrival of Valued Associate #679: Cesar Manara Planned maintenance scheduled April 23, 2019 at 00:00UTC (8:00pm US/Eastern)Random variable and probability space notionsRandom Walk with EdgesFinding functions where the increase over a random interval is Poisson distributedNumber of days until dayCan an observed event in fact be of zero probability?Unit random processmodels of coins and uniform distributionHow to get the number of successes given $n$ trials , probability $P$ and a random variable $X$Absorbing Markov chain in a computer. Is “almost every” turned into always convergence in computer executions?Stopped random walk is not uniformly integrable

            How should I support this large drywall patch? Planned maintenance scheduled April 23, 2019 at 00:00UTC (8:00pm US/Eastern) Announcing the arrival of Valued Associate #679: Cesar Manara Unicorn Meta Zoo #1: Why another podcast?How do I cover large gaps in drywall?How do I keep drywall around a patch from crumbling?Can I glue a second layer of drywall?How to patch long strip on drywall?Large drywall patch: how to avoid bulging seams?Drywall Mesh Patch vs. Bulge? To remove or not to remove?How to fix this drywall job?Prep drywall before backsplashWhat's the best way to fix this horrible drywall patch job?Drywall patching using 3M Patch Plus Primer